LSAT and Law School Admissions Forum

Get expert LSAT preparation and law school admissions advice from PowerScore Test Preparation.

User avatar
 Dave Killoran
PowerScore Staff
  • PowerScore Staff
  • Posts: 5853
  • Joined: Mar 25, 2011
|
#46195
Complete Question Explanation
(The complete setup for this game can be found here: lsat/viewtopic.php?t=6424)

The correct answer choice is (E)

If M is not interviewed, then from contrapositive of the fifth rule, K is not hired. Because from the third rule F must be interviewed, answer choice (E) must be true and is therefore correct.

Answer choice (A) is incorrect because although K cannot be hired, it is possible for K to be interviewed.

Answer choice (B) is incorrect because although K can be interviewed, K does not have to be interviewed.

Answer choice (C) is incorrect because it is possible for F to be hired.

Answer choice (D) is incorrect because while it is possible for F to be hired, F does not have to be hired.

Answer choice (E) is the correct answer choice as explained above.

Get the most out of your LSAT Prep Plus subscription.

Analyze and track your performance with our Testing and Analytics Package.